Đến nội dung

royal1534

royal1534

Đăng ký: 09-07-2015
Offline Đăng nhập: 20-12-2023 - 20:00
****-

#648032 CMR $\frac{x}{x\sqrt{x+yz}}+...

Gửi bởi royal1534 trong 05-08-2016 - 12:16

Cho ba số thực dương x, y, z thỏa mãn x + y + z = 1
CMR $\frac{x}{x\sqrt{x+yz}}+\frac{y}{y+\sqrt{y+zx}}+\frac{z}{z+\sqrt{z+xy}}\leq 1$

Áp dụng bất đẳng thức Cauchy-Schwarz ta có:

$x+\sqrt{x+yz}=x+\sqrt{x(x+y+z)+yz}=x+\sqrt{(x+y)(x+z)} \geq x+\sqrt{xy}+\sqrt{xz}$

Suy ra $\frac{x}{x+\sqrt{x+yz}} \leq \frac{x}{x+\sqrt{xy}+\sqrt{zx}}=\frac{\sqrt{x}}{\sqrt{x}+\sqrt{y}+\sqrt{z}}$

Thiết lập 2 bđt tương tự và cộng lại ta có đpcm. Dấu '=' xảy ra khi $x=y=z=\frac{1}{3}$




#647902 Tìm số nguyên dương n sao cho n chia hết cho mọi số nguyên dương không vượt q...

Gửi bởi royal1534 trong 04-08-2016 - 13:26

Tìm số nguyên dương n sao cho n chia hết cho mọi số nguyên dương không vượt quá $\sqrt{n}$




#647711 Chứng minh $\sum\frac{1}{(a+b+2\sqrt{...

Gửi bởi royal1534 trong 02-08-2016 - 23:51

Cho a,b,c dương thoả mãn 

$\frac{1}{a}+\frac{1}{b}+\frac{1}{c}\leq 16(a+b+c)$

Chứng minh

$\sum\frac{1}{(a+b+\sqrt{2(a+c)})^3}\leq\frac{8}{9}$

 

Hint

Áp dụng bất đẳng thức AM-GM ta có 

$(a+b+\frac{\sqrt{2(a+c)}}{2}+\frac{\sqrt{2(a+c)}}{2})^3 \geq \frac{27}{2(a+b)(a+c)}$

Suy ra $VT \leq \sum \frac{2}{27(a+b)(a+c)}$

Và ta quy bài toán về chứng minh $\sum \frac{1}{(a+b)(a+c)} \leq 12 $

$\Leftrightarrow (a+b+c) \leq 6(a+b)(b+c)(c+a)$

Giả thiết tương đương với $16(a+b+c)abc \geq ab+bc+ca$

Mà theo bđt AM-GM thì ta có: $(ab+bc+ca)^2 \geq 3(a+b+c)abc \geq \frac{3}{16}(ab+bc+ca)$

$\Leftrightarrow ab+bc+ca \geq \frac{3}{16}$

Áp dụng bổ đề quen thuộc trên (Chứng minh khá dễ). Ta có

$6(a+b)(b+c)(c+a) \geq 6.\frac{8}{9}.(a+b+c)(ab+bc+ca) \geq 6.\frac{8}{9}.\frac{3}{16}(a+b+c)=a+b+c $

Ta có Q.E.D. Dấu '=' xảy ra khi $a=b=c=\frac{1}{4}$




#647144 Cho x, y, z>0 : CMR: $\sum \frac{x^{3}...

Gửi bởi royal1534 trong 30-07-2016 - 08:48

À với cách đặt như vậy thì $abc=1$ nên em liên tưởng tới BĐT $vasc: \;\;\;\;\;\;\;\ \sum \frac{1}{a^{2k}+a^{k}+1}\geq 1.$

Ta sẽ chứng minh: $\frac{1}{(1+a)^3}\geq \frac{3}{8(a^{2k}+a^k+1)}\Leftrightarrow 8a^{2k}+8a^k+5\geq 3a^3+9a^2+9a$

Để có được như vậy thì đẳng thức phải xảy ra, tức là $8a^{2k}+8a^k+5= 3a^3+9a^2+9a$

Đạo hàm cả $2$ vế thì được $16k.a^{2k-1}+8k.a^{k-1}=9a^2+18a+9$

Mà dấu $"="$ xảy ra khi $x=y=z$ nên $a=1,$ từ đó tính được $k=\frac{3}{2}.$

Việc cuối cùng chỉ là đi chứng minh BĐT: $\frac{1}{(1+a)^3}\geq \frac{3}{8(a^3+\sqrt{a^3}+1)}$ là đúng.

---------------------------------------

Em cũng không biết tại sao anh lại nghĩ đến chứng minh BĐT: $(1+a)^3 \leq (1+abc)(1+\frac{a}{b})(1+\frac{a}{c})$

Chỉ là bài toán anh đọc qua rồi. Nhưng anh nghĩ mò mẩm để cho ra bđt quen thuộc là chính :~




#647064 Cho x, y, z>0 : CMR: $\sum \frac{x^{3}...

Gửi bởi royal1534 trong 29-07-2016 - 18:51

Lời giải:

Ta có bất đẳng thức cần chứng minh tương đương với:

$$\frac{1}{(1+\frac{y}{x})^3}+\frac{1}{(1+\frac{z}{y})^3}+\frac{1}{(1+\frac{x}{z})^3}\geq \frac{3}{8}\;\;\;\;\;\;\;\;\;\;\;\;\;\;\;\;\;\;\;\;\ (1)$$

Đặt $a=\frac{y}{x};b=\frac{z}{y};c=\frac{x}{z}$ thì $abc=1$ và $(1)$ viết lại thành:

$$\frac{1}{(1+a)^3}+\frac{1}{(1+b)^3}+\frac{1}{(1+c)^3}\geq \frac{3}{8}$$

Ta sẽ chứng minh:

$$\frac{1}{(1+a)^3}\geq \frac{3}{8(a^3+\sqrt{a^3}+1)}$$

$$\Leftrightarrow 8a^3+8\sqrt{a^3}+8\geq 3a^3+9a^2+9a+3$$

$$\Leftrightarrow 5a^3+8\sqrt{a^3}+5\geq 9a^2+9a\;\;\;\;\;\;\;\;\;\;\ (2)$$

Bất đẳng thức trên luôn đúng theo $\textrm{AM-GM}:$

  • $$3a^3+6\sqrt{a^3}=3(a^3+\sqrt{a^3}+\sqrt{a^3})\geq 3.3a^2=9a^2$$
  • $$2a^3+2\sqrt{a^3}+5=a^3+a^3+\sqrt{a^3}+\sqrt{a^3}+1+1+1+1+1\geq 9a$$

Cộng $(2)$ vế BĐT trên lại ta được $(2)$ đúng.

Chứng minh tương tự với $b,c$ suy ra:

$$\frac{1}{(1+a)^3}+\frac{1}{(1+b)^3}+\frac{1}{(1+c)^3}\geq \frac{3}{8(a^3+\sqrt{a^3}+1)}+\frac{3}{8(b^3+\sqrt{b^3}+1)}+\frac{3}{8(c^3+\sqrt{c^3}+1)}$$

Cần chứng minh:

$$\frac{1}{a^3+\sqrt{a^3}+1}+\frac{1}{b^3+\sqrt{b^3}+1}+\frac{1}{c^3+\sqrt{c^3}+1}\geq 1$$

Nếu đặt $\sqrt{a^3}=m; \sqrt{b^3}=n; \sqrt{c^3}=p$ thì BĐT này trở nên quen thuộc:

$$\frac{1}{m^2+m+1}+\frac{1}{n^2+n+1}+\frac{1}{p^2+p+1}\geq 1.$$

Chứng minh

Vậy ta có đpcm.

 

P/S

Lời giải của Khánh rất hay. Anh không biết sao mà em tìm được bđt trung gian như vậy

Một lời giải khác:

----------------

Ta cũng đưa bđt về chứng minh:

$\frac{1}{(1+a)^3}+\frac{1}{(1+b)^3}+\frac{1}{(1+c)^3} \geq \frac{3}{8}$

Áp dụng bđt Holder ta có

$(1+a)^3 \leq (1+abc)(1+\frac{a}{b})(1+\frac{a}{c})$

$\Leftrightarrow \frac{1}{(1+a)^3} \geq \frac{bc}{2(a+b)(a+c)}$ (Vì $abc=1$)

Thiết lập các bđt tương tự và cộng lại ta có
$\sum \frac{1}{(1+a)^3} \geq \frac{1}{2}[ \sum \frac{bc}{(a+b)(a+c)}] $

Vậy ta chỉ cần chứng minh 

$\frac{bc}{(a+b)(a+c)}+\frac{ca}{(b+c)(b+a)}+\frac{ab}{(c+b)(c+a)} \geq \frac{3}{4}$

Quy đồng và biến đổi tương đương. BĐT trên tương đương với 

$ab(a+b)+bc(b+c)+ca(a+c) \geq 6abc$

$\leftrightarrow b(a-c)^2+c(a-b)^2+a(b-c)^2 \geq 0$: Đúng

Ta có điều phải chứng minh. Dấu '=' xảy ra khi $a=b=c$




#647005 $(a-b)^2(b-c)^2(c-a)^2$

Gửi bởi royal1534 trong 29-07-2016 - 12:01

Cái đầu tiên là "thành viên VMF" anh ơi :V




#646977 Cmr:$a+b+c+\sqrt{3}\geq 8abc(\frac{1}...

Gửi bởi royal1534 trong 29-07-2016 - 06:36

Cho các số thực dương a,b,c thỏa mãn $ab+bc+ca\leq 1$. Chứng minh rằng:

$a+b+c+\sqrt{3}\geq 8abc(\frac{1}{a^2+1}+\frac{1}{b^2+1}+\frac{1}{c^2+1})$

Áp dụng bất đẳng thức AM-GM ta có
$a^2+1=a^2+\frac{1}{3}+\frac{1}{3}+\frac{1}{3} \geq 4\sqrt[4]{\frac{a^2}{27}}=4\frac{\sqrt{a}}{\sqrt[4]{27}}$
Tương tự:$b^2+1 \geq 4\frac{\sqrt{b}}{\sqrt[4]{27}}$
                $c^2+1 \geq 4\frac{\sqrt{c}}{\sqrt[4]{27}}$
Suy ra $VP \leq 8abc.\sum \frac{\sqrt[4]{27}}{4\sqrt{a}}=2\sqrt[4]{27}(\sum bc\sqrt{a})$
Áp dụng bất đẳng thức AM-GM với nhận xét $1 \geq ab+bc+ca$ ta có
$ab+bc+ca \geq 3\sqrt[3]{a^2b^2c^2} \geq 3\sqrt[3]{a^2b^2c^2}.\sqrt[3]{ab+bc+ca}$
$\Leftrightarrow \sqrt[3]{(ab+bc+ca)^2} \geq 3\sqrt[3]{a^2b^2c^2} $
$\Leftrightarrow (ab+bc+ca)^2 \geq 27a^2b^2c^2 $
$\Leftrightarrow \sqrt{ab+bc+ca} \geq \sqrt[4]{27}\sqrt{abc}$
$\Leftrightarrow \frac{\sqrt{ab+bc+ca}}{\sqrt[4]{27}} \geq \sqrt{abc}$
Suy ra: $\sum bc\sqrt{a} =\sqrt{abc}(\sqrt{bc}+\sqrt{ca}+\sqrt{ab}) \leq \frac{\sqrt{ab+bc+ca}}{\sqrt[4]{27}}.\sqrt{3(ab+bc+ca)}=(ab+bc+ca).\frac{\sqrt{3}}{\sqrt[4]{27}}$
Nên ta có $VP \leq 2\sqrt[4]{27}.(ab+bc+ca).\frac{\sqrt{3}}{\sqrt[4]{27}}=2\sqrt{3}(ab+bc+ca)$
Áp dụng bất đẳng thức AM-GM ta có
$VT=a+b+c+\sqrt{3} \geq 2\sqrt{\sqrt{3}(a+b+c)} \geq 2 \sqrt{3(ab+bc+ca)}$
Vậy ta quy bài toán về chứng minh:
$2\sqrt{3(ab+bc+ca)} \geq 2\sqrt{3}(ab+bc+ca) $
$\leftrightarrow 3(ab+bc+ca) \geq 3(ab+bc+ca)^2$
$\leftrightarrow (ab+bc+ca)(1-ab-bc-ca) \geq 0$: Đúng vì $0<ab+bc+ca \leq 1$
Ta có Đpcm. Đẳng thức xảy ra khi $a=b=c=\frac{1}{\sqrt{3}}$




#646555 $Max P= \frac{1}{2x^2+z}+\frac{1...

Gửi bởi royal1534 trong 26-07-2016 - 13:29

Cho $x,y,z$ là các số thực thõa $1 \leq x,y,z \leq 2$

Tìm GTLN của biểu thức $P= \frac{1}{2x^2+z}+\frac{1}{2y^2+z}-\frac{2xy+1}{xy(z+2)} $




#645762 $P = \sqrt{5x^2+xy+3y^2}+ \sqrt{3x^2+xy+5y^2...

Gửi bởi royal1534 trong 21-07-2016 - 02:48

Phương pháp gì để đoán được lượng đại diện vậy anh

Em sử dụng công thức tổng quát sau:

$ma^2+nb^2+pab=\frac{4mn-p^2}{4(m+n+p)}(a-b)^2+\frac{[a(2m+p)+b(2n+p)]^2}{4(m+n+p)}.$

Từ đó có thể làm trội lên vì $(a-b)^2 \geq 0$  :D

-------------
Bài trên có thể dùng bđt Mincopxki giải tự nhiên hơn  :lol:




#645594 $P = \sqrt{5x^2+xy+3y^2}+ \sqrt{3x^2+xy+5y^2...

Gửi bởi royal1534 trong 19-07-2016 - 23:02

Ta chứng minh được một loạt các bđt sau bằng biến đổi tương đương:

$\sqrt{5x^2+xy+3y^2} \geq \frac{7y+11x}{6} \Leftrightarrow (x-y)^2 \geq 0$

$\sqrt{3x^2+xy+5y^2} \geq \frac{7x+11y}{6} \Leftrightarrow (x-y)^2 \geq 0$

Tương tự : $\sqrt{x^2+xy+2y^2} \geq \frac{3x+5y}{4}$

                   $\sqrt{2x^2+xy+y^2} \geq \frac{5x+3y}{4}$

Cộng các bất đẳng thức vừa tim được ta có

$VT \geq \frac{7x+11y+11x+7y}{6}+\frac{3x+5y+5x+3y}{4}=5(x+y)=5.2016=10080$

Dấu '=' xảy ra khi $x=y=1008$




#645203 $\frac{a+b+c+\sqrt[3]{abc}}{4}...

Gửi bởi royal1534 trong 16-07-2016 - 20:24

Cho $a,b,c$ là các số thực dương. Chứng minh rằng:

$$\frac{a+b+c+\sqrt[3]{abc}}{4}\geq \frac{\sqrt{ab}+\sqrt{bc}+\sqrt{ca}}{3}.$$

Đặt $\sqrt[3]{a}=x,\sqrt[3]{b}=y,\sqrt[3]{c}=z$

Ta cần chứng minh: $3(a^3+b^3+c^3+abc) \geq 4ab\sqrt{ab}+4bc\sqrt{bc}+4ca\sqrt{ca}$

Sử dụng bất đẳng thức AM-GM. Áp dụng liên tục 2 bđt sau:

1.$ a^3+b^3 \geq ab(a+b) \geq 2ab\sqrt{ab} $

Tương tự $\Rightarrow 2(a^3+b^3+c^3) \geq 2ab\sqrt{ab}+2bc\sqrt{bc}+2ca\sqrt{ca}$

2.$ a^3+b^3+c^3+3abc \geq ab(a+b)+bc(b+c)+ca(a+c) \geq 2ab\sqrt{ab}+2bc\sqrt{bc}+2ca\sqrt{ca}$ (BĐT Schur)

Cộng 2 bất đẳng thức vừa tìm được ta có đpcm. Dấu '=' xảy ra khi $x=y=z$ hay $a=b=c$




#645118 Cho các số thực $a,b$ thỏa mãn: $a^2+b^2\le 1$. Chứn...

Gửi bởi royal1534 trong 16-07-2016 - 03:53

Xét $c^2+d^2 \geq 1$. Kết hợp $a^2+b^2 \leq 1$: Ta có $VT \geq 0 \geq VP$ (Đpcm)

Xét $c^2+d^2 \leq 1$.Kết hợp giả thiết $a^2+b^2 \leq 1$:

Áp dụng bđt Cauchy ta có $2=a^2+b^2+c^2+d^2 \geq 2(ac+bd) \leftrightarrow 1 \geq ac+bd$

BĐT cần chứng minh tương đương với

$|ac+bd-1| \geq \sqrt{(1-a^2-b^2)(1-c^2-d^2)}$

$\leftrightarrow 1-ac-bd \geq \sqrt{(1-a^2-b^2)(1-c^2-d^2)}$

Áp dụng bđt AM-GM ta có :

$\sqrt{(1-a^2-b^2)(1-c^2-d^2)} \leq \frac{2-a^2-b^2-c^2-d^2}{2} \leq \frac{2-2ac-2bd}{2}=1-ac-bd$

Ta có điều phải chứng minh. Dấu '=' xảy ra khi $a=b=c=d=\frac{1}{\sqrt{2}}$




#644705 Chứng minh $\sum \frac{a}{\sqrt{(b+c)...

Gửi bởi royal1534 trong 12-07-2016 - 18:47

Gọi P là biểu thức VT và đặt:

$S=a[(b+c)^2+5c^2]+b[(c+a)^2+5a^2]+c[(a+b)^2+5b^2]$.

Sử dụng BĐT Holder ta có: $P^2.S\geq (a+b+c)^3$.

Vậy ta chứng minh:

$(a+b+c)^3\geq S\Leftrightarrow a(a-b)^2+b(b-c)^2+c(c-a)^2\geq 0$.

BĐT cuối luôn đúng.

Dấu bằng xảy ra khi $a=b=c$.

Bạn nhầm rồi thì phải. 

BĐT cần chứng minh là $(a+b+c)^3 \geq \sum a[(b+c)^2+5c^2] \leftrightarrow a^3+b^3+c^3+2(a^2b+b^2c+c^2a) \geq 3(ab^2+bc^2+ca^2)$
BĐT trên không tương đương với bđt cuối cùng của bạn !




#644296 CMR: $(a^2+2)(b^2+2)(c^2+2) \geq 3(a+b+c)^2+(abc-1)^2$

Gửi bởi royal1534 trong 10-07-2016 - 02:25

Cho a,b,c dương. Chứng minh rằng:

$(a^2+2)(b^2+2)(c^2+2) \geq 3(a+b+c)^2+(abc-1)^2$

Sử dụng một bổ đề quen thuộc sau:  
 

Với $a,b,c$ dương ta có: $a^2+b^2+c^2+2abc+1 \geq 2(ab+bc+ca)$

 

------------------------------
Thực hiện phép khai triển. BĐT đã cho tương đương với: 

$a^2+b^2+c^2+2abc+1+2(a^2b^2+b^2c^2+c^2a^2)-6(ab+bc+ca)+6 \geq 0$

 

Áp dụng bổ đề trên. Ta có
 

$VT \geq 2(a^2b^2+b^2c^2+c^2a^2)+2(ab+bc+ca)-6(ab+bc+ca)+6=2(ab-1)^2+2(bc-1)^2+2(ca-1)^2 \geq 0$ 
 

Ta có điều phải chứng minh. Đẳng thức xảy ra khi $a=b=c$
 




#643548 1)Cho 3 số thực dương a,b,c thỏa mãn a+b+c=3.CMR: $\sum \frac...

Gửi bởi royal1534 trong 04-07-2016 - 01:00

1)Cho 3 số thực dương a,b,c thỏa mãn a+b+c=3.CMR:

$\sum \frac{1}{a^{2}}\geq \sum a^{2}$

BĐT cần chứng minh $\Leftrightarrow \sum \frac{1}{a^2} \geq 9-2\sum ab$
                                   $\Leftrightarrow \sum \frac{1}{a^2}+2\sum ab \geq 9$
Áp dụng bất đẳng thức AM-GM ta có:
$(ab+bc+ca)^2 \geq 3abc(a+b+c)=9abc$
$\sum \frac{1}{a^2} \geq \frac{(\sum \frac{1}{a})^2}{3}=\frac{(ab+bc+ca)^2}{3a^2b^2c^2}  \geq \frac{9abc}{3a^2b^2c^2}=\frac{3}{abc} \geq \frac{27}{(ab+bc+ca)^2}$
Khi đó. Bằng phép đặt $ab+bc+ca=t$

Ta quy bài toán về chứng minh $\frac{27}{t^2}+2t \geq 9$ (Đúng theo AM-GM vì $\frac{27}{t^2}+t+t \geq 3\sqrt[3]{27}=9$)
Ta có đpcm. Đẳng thức xảy ra khi $a=b=c=1$